Sei sulla pagina 1di 46

Products Liability

Subspecies of common law torts w/ significant portions of contract law 1916: McPherson v. Buick Motors demise of privity requirement in negligence 1963: court 1st imposed strict liability in a product injury case Law of product liability has been packed into the last century a very modern area of law Response to tremendous increase in product injury cases in the early 20th century If products are not carefully made, they have the potential for creating significant #s of product injuries Roger Trainer author of 1st strict liability case Focus on need for something more than existing tort & contract law Pages 3 & 4 of the text: lists of variety and numbers of products A hazard is unconscionable when: 1) When consumer does not know the harm exists 2) Aware of risk but not of its frequency and severity 3) Consumer does not know how to cope with risk 4) The risk could be eliminated by increase in cost that consumers would pay if they knew of this option Behaviors of consumers Environment it is used in Condition of product sold Focus on product manufacturers want to encourage manufactures to make better products No motivation for manufacturers to make better products in the beginning No privity Significant incentive to sue for product liability now strict liability was often found & huge jury awards this eventually led to a crisis where legal commentators took a second look at strict liability lots of tort reform efforts were focused on products liability in the 80s and there was a shift away from products liability in many jurisdictions, at least in a way that made it seem less onerous for manufacturers (state law is applied [Always? Usually?] but these cases often land in federal court under diversity jurisdiction)

PRODUCTS LIABILITY 2009 Matthew J. Weaver

Page 1

Risk retention act helps manufacturers pool their resources to get product liability resources Product liability has 2 distinct areas: 1) Public law concerned w/ product safety primarily regulatory and addresses prevention (ex ante) we arent talking about this in this class 2) [I spaced out but the second one is what were talking about for this class] Theories of manufacture liability: 1. negligence 2. misrepresentation (tortuous) 3. breach of warranty (regards a form of misrepresentation) (UCC sections 1 & 2) 4. strict liability Concept of defect: 1) manufacturing 2) design 3) marketing (warning/informational defect) ^^^ Under each theory, we will address each type of defect! ^^^ Then, when we talk about the concept of defect, we will talk about the various types of defects Manufacturing = almost always strict liability, everywhere Design & warning (she called marketing warning) = not usually strict liability MacPherson v. Buick Motors page 50 * RULE: A manufacturer of a product which, if negligently constructed is reasonably certain to be dangerous to life, is liable to indirect retail purchasers for injuries caused by such negligent construction. * INFO: This landmark case gives expression to law as it remains even today. Later developments in this area extended liability to users of the products who were not purchasers at all, and in appropriate cases, to bystanders as well. A duty to inspect a product encompasses a duty to inspect the component parts of such a product, and where liability attaches, it cannot be avoided by alleging purchase of a component from another manufacturer. the attack on the citadel of privity What is privity? o The immediate relationship between the buyer and seller o Only those who have an actual face-to-face relationship are responsible to one another in product disappointment cases Product liability = basically product disappointment (so disappointing that the customer is injured) o In the olden days, relationships were simple: immediate relationship between the manufacturer of the product and the purchaser of the product PRODUCTS LIABILITY 2009 Matthew J. Weaver Page 2

Now, the relationship between the consumer and the person who made the product is much more distant. Now, it goes something like this: manufacturer wholesaler retailer consumer o Witterbottom v. Wright [sp?] British stagecoach case: stagecoach broke & in a famous case, the court denied recovery because the parties werent in privity the court didnt want to open up the floodgates of litigation back then, privity requirement was still a barrier for the injured plaintiff who wanted to get to a remote manufacturer o Exceptions: Act of negligence posed an imminent danger to life and health (most commonly asserted because its the easiest to prove limited to things like firearms, explosives, poisons, drugs things that you could clearly see that will be inherently dangerous or if they are defectively manufactured or if information isnt adequately provided) Act of negligence caused injury to one invited onto the defendants premises An imminently dangerously article was delivered by a defendant who knew about the danger of the product and knew that the recipient didnt know What happened in MacPherson v. Buick Motors: o Wheel collapsed and injury occurred Buick didnt make the wheel The wheel was a component (so no privity under old law) o Cordozo says that you dont have to be in privity of K o How does Cordozo frame the issue? What is the duty owed by the manufacturer of the car to the purchaser of the car, a consumer who bought the product through an intermediary? Buick had a duty to inspect the wheel and make sure that it was made carefully o Basic holding of the case: The manufacturer should know that the risk of harm is not on the immediate purchaser (the car dealership) but rather the consumer (the person who will be driving the car) or other people on the road This courts assessment of privity can be compared to the way the issue in Palsgraf was framed: Who is within the zone of danger created by the risk? o Cordozo took many of the imminent danger cases that dealt w/ poisons & explosions, which were exceptions to the privity rule, and got rid of them making this general rule: if a risk is posed to a 3rd party by a manufacturers product, the manufacturer can be held liable for a 3rd partys harm caused by the product

PRODUCTS LIABILITY 2009 Matthew J. Weaver

Page 3

The purpose of discussing the different theories that existed before is to help us recognize/assess the inadequacies of the old system Part of the reason why sweeping strict liability was enacted: the feeling that negligence & warranty were not adequate remedies We have seen an emergence of negligence law as a significant theory many states have went back to the negligence approach and RST 3rd of torts calls everything strict liability but if you read closely, you can see that it often is actually speaking of negligence Generic risks v. non-generic risks = what this means is that, for the most part: Non-generic: In a manufacturing defect case, we only have 1 product or 1 batch of products that are the problem (i.e. somebody didnt screw in a bolt right that day, or there was a dull cutting tool so a batch had a problem but all other products had no problems) Generic: every product sold without proper warnings or instructions are defective or is improperly designed is defective o Negligence = Conduct falling below the standard of care that a reasonable person would demonstrate under similar conditions. Jenkins v. General Motors: * RULE: A jury may make a finding of negligence even though expert opinion differs regarding causation. * INFO: As this case indicates, products liability cases are often extremely complex, requiring expert witnesses, photographs, exhibits, and films for the jury. The use of expert testimony is often crucial to winning a products liability case. Thus, most attorneys who regularly practice in this area seek expert witnesses who not only are knowledgeable, but also make credible and effective witnesses. Moving car malfunctions and a passenger is paralyzed permanently o Passenger sues the manufacturer of the car P says that nuts and bolts werent properly attached on the cars suspension D (General Motors) states that the nuts and bolts were properly attached and were knocked off because of the accident o Case goes to the jury and the jury finds for the P o D appeals, saying that there was not enough evidence for the case to go to the jury On appeal, the judge says that the evidence was equiponderant which means that the evidence did not favor either party it was a battle of the experts Holding: when the evidence of causation comes down to a battle of the experts it is proper to take the case to the jury Was negligence an issue here? o The court doesnt talk about negligence the court lets the jury infer from the experts kind of like res ipsa loquitur but res ipsa is used when the PRODUCTS LIABILITY 2009 Matthew J. Weaver Page 4

court cant prove a specific act of negligence, but here the plaintiff comes up with an act of negligence (products liability tort) Assignment for Week of January 12 Products Liability and Safety pp. 1-15, 41-45, 50-59, 59-69, 69-75, 76-90 Ford Motor Company v. Zahn Pg. 56 * RULE: A manufacturer has a duty to exercise reasonable care to prevent defective conditions caused by a miscarriage in the manufacturing process. * INFO: Prosser, in his treatise, states that: "Negligence is a matter of risk that is to say, of recognizable danger of injury. It has been defined as `conduct which involves an unreasonably great risk of causing damage,' or, more fully, conduct `which falls below the standard established by law for the protection of others against unreasonably great risk of harm.'" Prosser, Torts, 2d Ed. 119. (quality control ashtray mistake takes out eye) in 1956, Zahn was riding in the passenger seat of a new Ford car o Zahn dropped his cigarette lighter on the floorboard, and when he reached down to get it, the driver had to break really hard this threw Zahn forward and his eyeball hit the ashtray in the dashboard The ashtray had a jagged burr on it and it cut Zahn up pretty bad (he lost his eye) Expert testimony showed that the edge on the ashtray was jagged because the cutting die that made it was dull Zahn brought his suit under a negligence theory he claimed that the sharp edge of the ashtray was a manufacturing defect that caused his injuries o The trial court jury found for Zahn. Fords judgment for judgment notwithstanding the verdict was denied. Ford then appealed, contending that it had properly inspected the ashtray and the type of harm Zahn suffered was not foreseeable Issue: does a manufacturer have a duty to exercise reasonable care to prevent a defective product from being released to the public, when the defect originates from the manufacturing process? o Holding: Yes. There is imposed on the manufacturer of an article for sale or use the duty to exercise reasonable care to prevent defective conditions caused by a miscarriage in the manufacturing process. This duty requires reasonable skill and care in the process of manufacture and for reasonable inspection or tests to discover defects. The jagged burr was readily discoverable. If Ford would have inspected all of the ashtrays, instead of just a sampling of them, the defect would have been discovered. Ford argued that the foreseeable harm was limited to snagged clothing or a cut finger, and not the loss of an eyeball. The court rejects this line of reasoning, finding PRODUCTS LIABILITY 2009 Matthew J. Weaver Page 5

that it was foreseeable that a passenger might be thrown forward toward the dashboard when the driver has to break The higher the risk & the greater the possible injury, the higher the quantum of care

Playschool pg 60 Hand formula page 63 Mesman 66 Jenkins contd: o Problems: When a product is damaged or destroyed in the accident, it is often difficult to determine what happened in these cases, we often rely on res ipsa

Strict liability has had the most lasting effect in defective product cases In what circumstances do we allow plaintiff to rely on res ipsa loquitur? What are important factors to consider? The actions of the plaintiff or a third party at the time of the accident The post-sale condition of the product (Was the product poorly maintained? Did the car have hundreds of thousands of miles on it?) Hand formula: 1) likelihood that this conduct will injure others, taken with 2) the seriousness of injury if it happens, balanced against 3) the interest which he must sacrifice to avoid the risk (B<PxL=N) In negligence, before we get to the application of the calculus of risk, what we have to have is a foreseeable risk Probability of harm occurring is low, but the enormity of the possible harm Metzgar v. Playskool (negligent design when kid chokes on regulatory agency-compliant block) child chokes on a block issue: adequacy of the design of the product for its foreseeable uses and users trial court says that the risk was foreseeable, but statistically the risk of harm to the foreseeable users was so small that it did not surpass the realistic threshold of risk o BUT: Court of appeals says that the risk was foreseeable and remote, but the jury should have heard the case (rather than the court granting summary judgment to the defendant)

PRODUCTS LIABILITY 2009 Matthew J. Weaver

Page 6

Reasonable alternative design: what would the defendant have to do to eliminate the risk caused? Compliance with a regulatory standard: compliance with a regulatory standard may be strong evidence in favor of a defendant maintaining its duty of care, but it is not disparities of the case Federal pre-emption: [I spaced out, but this is important]

Mesman v. Konecranes (Indiana case poorly refurbished crane partially smashes a guy negligent design when foreseeable that a worker would press the wrong button) * RULE: The risk of an injury must be weighed against the cost of avoiding the injury in assessing whether a failure to take precautions in product design was negligent. * INFO: In Mesman, the Seventh Circuit applies the formula for assessing negligence announced in United States v. Carroll Towing Co., (2d Cir. 1947). The appellate court rules here that a reasonable jury could have seen Konecranes's (D) failure to take greater precaution in the rebuilt cab as negligent. Boyl v. California Chemical Company (marketing/duty to warn: arsenic-based weed killer gives lady resting in grass chemical burns defendant liable) burden of prevention was simply adding a few words to the warning * RULE: A manufacturer of dangerous products must provide warnings for all incidental and attendant uses. Week of January 18 pp. 76 - 90, 91 - 102, 102 111 In class January 21: This next set of materials we will be talking addresses the problem of the provision of false information about a product what was said about the product this is referred to generally as Tortious Misrepresentation On the board: What is required to establish FRAUD: (must fulfill all of these elements) 1) Uttering of false statement of present material fact 2) Knowing it to be false (scienter requirement) 3) Intent to induce reliance 4) Reliance: the other partys justifiable reliance 5) Damage to other party Fraud: not a favored theory because of its specific requirements * Important points about fraud: When economic losses are involved in a products liability case, look to a theory of fraud first (when youre taking the test, that is). Also: Under a fraud theory, privity does not need to be established.

PRODUCTS LIABILITY 2009 Matthew J. Weaver

Page 7

St. Joseph Hospital v. Corbetta Construction Company page 76 * RULE: A technically true statement may nevertheless be fraudulent when it does not state matters that materially qualify the statement as it was made. * In class: Strictly speaking, the statement made was not false, but it was misleading. Under a fraud theory, the accused partys intent must be to mislead the other party by inducing reliance through false statements. Another note: This is an economic loss case only most cases we deal with in this class involve physical injury. The plaintiffs would probably have not recovered under any other theory, since this case involves economic damages only. Baxter v. Ford Motor Company page 83 * RULE: A manufacturer or retailer of a product is responsible in tort for all representations upon which the customer must rely, regardless of the presence or absence of a contractual relationship between the plaintiff and defendant. * In class: A MacPherson type of case, but here we are dealing with something that has been said about the product MacPherson dealt with conduct, not a manufacturers statements. This case was probably originally put forth under a theory of Violation of Express Warranty (an action in Contract) but a privity issue would exist under this theory. This was a representation made to the consumer that could not be verified. What this case did was create a Tort version of the Violation of Express Warranty theory. This case is an example of a Strict Liability situation proof of fault is not required. * Restatement 2nd of Torts 402B: Misrepresentation by Seller of Chattels to Consumer page 84: Must have physical harm under a RS Torts 402B claim. Physical harm must be caused by consumers justifiable reliance on sellers public misrepresentation of material fact concerning character or quality of product. Hauter v. Zogarts (I) page 85 * RULE: A misrepresentation of material fact upon which a person relies is actionable in tort. * In class: This is a RS 2nd Torts 402B claim. Statement on Golfing Gizmo box said, COMPLETELY SAFE BALL WILL NOT HIT PLAYER and ball hit player. Justifiable Reliance issue: kids mom bought the product and the kid (the hit player) read the box. Would there be any difference if the kid didnt read the box? Probably not, because he was so young. Would there be any difference if the mom read the box after she bought it, but before the accident? As long as the justifiable reliance has an impact on subsequent action, it is sufficient. * Restatement 3rd of Torts 9: Similar to RS 2nd Torts 402B, but it doesnt have to be a seller making the misrepresentations and a few other details are changed. The professor hasnt seen any cases brought under RS 3rd Torts 9 RS 3rd Torts as a whole might not be that influential. Warranty cases: Kolarik v. Cory Intl Corp. page 91 * RULE: The words minced pimento stuffed on a jar of olives do not constitute an express warranty that the olives have been pitted. PRODUCTS LIABILITY 2009 Matthew J. Weaver Page 8

* UCC 2-313: Express Warranties by Affirmation, Promise, Description, Sample page 93: (1) a) any affirmation of fact or promise made by seller to buyer becomes the basis of the bargain; b) any description of the goods which is the basis of the bargain creates an express warranty that the goods will conform to the description; c) any sample or model which is part of the basis of the bargain creates an express warranty that the goods shall conform to the sample or model. (2) It is not necessary for the creation of an express warranty that the seller use formal words such as warrant or guarantee or that he have a specific intention to make a warranty however, an affirmation merely of the value of the goods or a statement purporting to be the sellers opinion of the goods does not create a warranty. * INFO: The challenged language in Kolarik ("minced pimento stuffed") did not warrant that every olive was perfectly, entirely pitted. Compounding the linguistic weakness of the warranty claim on the merits here was the procedural, evidentiary problem: Kolarik's (P) own evidence agreed with Cory's (D) that pitting was not a perfect process. With no genuine issue of material fact on that point requiring trial, the issue thus could be resolved as a question of law on summary judgment. * In class: Warranty actions drawbacks: grew from the law of Contract and Sales, originally aimed at addressing problems in an economic business environment, not personal injury claims. Notice requirements under UCC. Differences in statute of limitations. Inconsistencies. Sellers right under the UCC to disclaim warranties, such as Merchantability or Fitness for a Particular Purpose. * In class: about the case: Whose expectations should a statement about a product address? The expectations of the industry the statement must be construed in light of trade standards. Examples: Does shelled nuts mean absolutely no shells in this product? Probably not. Does No Bones! mean boneless? Hell no! Express Warranty v. Strict Liability Tortious Misrepresentation SLTM requires representation to the public; express warranty requires that the statement be part of the basis of the bargain (may be formed by statement to individual buyer) SLTM requires physical harm; express warranty does not SLTM requires reliance; express warranty: some courts take the position that reliance is not required some courts take the position that reliance is required theres usually clear evidence of actual reliance, like in Hauter Stang v. Hertz Corp. page 96 * RULE: An affirmation of fact creates an express warranty if it becomes part of the basis of the bargain. * Reliance: this court says that reliance is part of UCC 2-313 (interpreting basis of the bargain to mean relied upon) follows interpretation from the Uniform Sales Act, which required reliance * INFO: The Uniform Commercial Code would seem to be in agreement with the holding in this case. One pertinent section provides that: "It is not necessary to the creation of an express warranty that the seller use formal words such as `warrant' or `guarantee' or that he have a specific intention to make a warranty, but an affirmation PRODUCTS LIABILITY 2009 Matthew J. Weaver Page 9

merely of the value of the goods or a statement purporting to be merely the seller's opinion or commendation of the goods does not create a warranty." U.C.C. 2-313(2). Hauter v. Zogarts (II) page 97 * RULE: A sellers representation regarding the safety of a product become part of the basis of the bargain and will be construed as an express warranty. *INFO: This case illustrates the impact that any statement by a seller can have on the seller's potential liability. Even a statement of opinion can become a warranty under the California U.C.C. 2313. Protection for a purchaser under this section is greater than that under both the former Uniform Sales Act and the Restatement of Torts. * Reliance: this court says that reliance is not part of UCC 2-313 * Think about it: most things that you buy dont have the express warranties printed on the outside of the box usually the express warranty terms are so complex that they have to be included in a pamphlet inside the box * Burden shifts from plaintiff to show reliance to defendant to show lack of reliance under Hauter * Other jurisdictions, including Indiana, have adopted another view on reliance: These courts say that reliance is not in the picture anymore plaintiff doesnt have to prove it & defendant does not have to disprove it. Maybank v. S.S. Kresge page 107 * RULE: For a product to be merchantable under UCC 2-314, the product must be both suitable and safe for its intended purpose. * UCC 2-314: Implied Warranty: Merchantability; Usage of Trade page 104 * Implied Warranty of Merchantability: exploding flashcube hurts a guys eye the flashcube wasnt fit for ordinary purposes for which such goods are used the classic shorthand referent for UCC 2-314 * To some extent, a failure to operate might not, in and of itself, breach UCC 2-314 but in this case, the product was not safe (the warning said that the product could shatter, but the warranty didnt say that the product could explode, which it did) * If something is sold AS IS this means that the seller is disclaiming UCC 2-314 * Under UCC 2-314, Plaintiff must prove these elements: that 1) merchant sold goods 2) which were not merchantable at the time of the sale, 3) injury and damages to plaintiff or its property 4) which were caused proximately and in fact by the defective nature of the goods, and 5) notice to the seller of injury * INFO: A plaintiff is often faced with a situation where there has been an injury producing failure or malfunction, without being able to pinpoint a cause for the failure. By bringing suit under a U.C.C. 2-314 warranty theory, the plaintiff can overcome these proximate cause problems. His burden is satisfied by proving the fact of the malfunction itself; there is no need to prove manufacturer's negligence in either the design or manufacture of the product. Assignment for Week of January 25 Products Liability and Safety pp. 111 - 114, 115 - 125, 126 - 151, 152 - 174 PRODUCTS LIABILITY 2009 Matthew J. Weaver Page 10

Barb v. Wallace page 111 * RULE: Whether an implied warranty of fitness for a particular purpose exists in any given case is a question of fact to be determined by the circumstances of the case. * UCC 2-315: Implied Warranty: Fitness for Particular Purpose page 111 * Under UCC 2-315, the seller does not have to be a merchant * Buyer has to be relatively ignorant about the product & must reasonably rely on the sellers skill or judgment to select or furnish suitable goods * Under UCC 2-315, particular purpose means specific to the use that the seller knows the buyer will use the product for (poorly worded but you know what I mean) fit for a particular purpose is a narrower requirement than merchantable the seller has to know the buyers purpose of use for the goods and the seller has to know that the buyer is reliant on the sellers representations * INFO: There is some overlap between the implied warranty of merchantability and the implied warranty of fitness for a particular purpose, and the courts are split as to whether a UCC 2-315 action will lie where one under 2-314 does. Note that there is also overlap between the implied warranty for a particular purpose and a product's express warranties. PRIVITY: Lukwinski v. Stone Container Corporation page 119 * RULE: Uniform Commercial Code-Sales 2-318 is not intended to allow recovery by all persons foreseeably affected by defects in the seller's product. * UCC 2-318: Third Party Beneficiaries of Warranties Express or Implied page 118 * INFO: In Lukwinski, the court, noting that it had the discretion to enlarge the scope of 2-318 where the circumstances of a case warrant, made clear that it was extremely reluctant, in light of the express wording of the statute, to expand its operation absent a compelling and well-founded reason. If coverage is not provided to employees of a corporate buyer, any warranties of the seller would be ineffective and extend to no person since it is impossible for a corporation to be the beneficiary. In the instant case, Lukwinski (P), other than asserting the need of protecting the general safety of delivery persons, provided to the court no compelling basis under warranty law for the extension of third-party beneficiary coverage under the facts of his case. Nevertheless, note that a plaintiff such as Lukwinski still does have a remedy for his injuries through other legal theories. Henningsen v. Bloomfield Motors, Inc. page 126 * RULE: The disclaimer of an implied warranty of merchantability imposed by law in transactions for the sale of goods is violative of public policy and void. * INFO: Unlike negligence liability, the defendant in a strict products liability case, who is otherwise without fault, may be held liable for merely selling a defective product. In the above case, for example, Mrs. Henningsen (P) was able to establish liability by testifying that she heard a loud noise, something cracked, and then the steering wheel spun in her hands, sending the car out of control. This was sufficient for the jury to infer

PRODUCTS LIABILITY 2009 Matthew J. Weaver

Page 11

that the car was defective, and that the defect occurred during the manufacturing process, although it was insufficient to infer negligence.

Dorman v. International Harvester Co. page 131 * RULE: In order to have a valid disclaimer provision, it must be in clear and distinct language, and prominently set forth in large, bold print in such position as to compel notice. * UCC 2-316: Exclusion or Modification of Warranties page 130 This section is designed principally to deal with those frequent clauses in sales contracts which seek to exclude all warranties, express or implied. It seeks to protect a buyer from unexpected and unbargained language of disclaimer by denying effect to such language when inconsistent with language of express warranty and permitting the exclusion of implied warranties only by conspicuous language or other circumstances which protect the buyer from surprise. McCarty v. E.J. Korvette, Inc. page 139 * RULE: Reasonable agreements which limit or modify remedies will be given effect, but the parties are not free to shape their remedies in an unreasonable or unconscionable way. * UCC 2-719: Contractual Modification or Limitation of Remedy page 138 The section of the Uniform Commercial Code dealing with contractual modifications or limitations of remedies states that: "Consequential damages may be limited or excluded unless the limitation or exclusion is unconscionable. Limitation of consequential damages for injury to the person in the case of consumer goods is prima facie unconscionable but limitation of damages where the loss is commercial is not." UCC 2-719(3). The present case is in accordance with this general rule. Ford Motor Co. v. Moulton page 143 * RULE: Implied warranties of merchantability and fitness may be disclaimed if the disclaimer is conspicuous and mentions merchantability. * INFO: The UCC holds that "it is of the very essence of a sales contract that at least minimum adequate remedies be available." In this case, the court concluded that those minimum remedies were in fact available, despite the disclaimer of implied warranties. The time limitation on the express warranties involved a fairly standard time period for automobile warranties. Implied warranty of merchantability: obligation to sell merchantable product created by operation of law, independent of parties agreement. Limitations on ability of seller to disclaim warranty or limit remedy: Henningsen: unconscionable for seller to disclaim warranties and limit remedies for personal injury under the circumstances of that case UCC 2-719(3): limitation of consequential damages for personal injury is prima facie unconscionable PRODUCTS LIABILITY 2009 Matthew J. Weaver Page 12

Greenman v. Yuba: * RULE: A manufacturer is strictly liable in tort when an article he places on the market, knowing it will not be inspected for defects, proves to have a defect that causes injury to a human being. * INFO: An imperfection in a product is not necessarily a defect. What constitutes a defect is a matter of reasonable consumer expectation. Furthermore, if a defect or a danger is very obvious, the product may be regarded as nondefective or, in appropriate cases, the manufacturer will be held only to a duty to warn of possible danger. A manufacturer is strictly liable on tort when an article he places on the market, knowing that it is to be used without inspection for defects, proves to have a defect that causes injury to a human being. Why? Manufacturer is in a better position than the consumer to pay the costs Restatement = not crafted to be law, unlike UCC ALI makes the Restatements Before Greenman, strict liability in tort had not been adopted by any state in the country Restatement 2d Torts 402A: One who sells a product in a defective condition unreasonably dangerous to the consumer is strictly liable if: The seller is in the business o selling the product The product was expected to and did reach the consumer without substantial change in condition Liability exists even though: Seller has used all possible care, and User has not entered into contractual relationship w/ seller (makes clear that the privity requirement of liability is gone!) Phipps v. General Motors: * RULE: In order to prove a cause of action for strict liability, the plaintiff need not prove any specific act of negligence on the part of the seller. * INFO: The reasoning of the court in adopting the rule of strict liability was that proof of a defect in a product at the time it left the control of the seller implies fault on the part of the seller sufficient to justify imposing liability for injuries caused by the product. Where the seller supplies a defective and unreasonably dangerous product, the seller or someone employed by him is held to have been at fault in designing or constructing the product. The court adopts strict liability. Strict liability v. warranty: strict liability cant be disclaimed, and remedies for personal injury cannot be limited. (True of both consumer & commercial products) & No requirement of privity for SL (foreseeability remains a concern) & no requirement of notice to the seller for SL, when suing under warranty theory Elements of strict liability (from Phipps): 1) Product is in defective condition when it leaves sellers control 2) Product is unreasonably dangerous to user or consumer 3) Defect caused injury

PRODUCTS LIABILITY 2009 Matthew J. Weaver

Page 13

4) Product expected to and did reach consumer in substantially unchanged condition Under negligence theory: P has to prove that what was wrong w/ the product was to be the fault of the manufacturer this is hard to prove, especially in modern times under SL, P does not have that burden There are defenses to negligence that courts refuse to extend to strict liability cases First National Bank page 182 * RULE: The imposition of strict liability causes manufacturers to make safer products. * INFO: Products liability law has become something of a political battleground in recent years. Supporters of the status quo point to increased product safety. Supporters of reform note that products liability laws have greatly increased costs of doing business and have hurt some industries. Strict liability from a perspective of protecting consumers a notion of deterrence Under a SL standard, manufacturers are encouraged to make better products For Monday, February 9, and Wednesday, February 11, please read pp. 156-167 n.2 and 171 n.5-225 of the text. * RST 2d 402A: Strict Liability: One who sells a product in a defective condition unreasonably dangerous to the consumer is strictly liable if: The seller is in the business of selling the product & the consumer was expected to and did reach the consumer without substantial change in condition. (Page 160) Liability exists even though seller has used all possible care and even though user has not entered into contractual relationship with seller. * Why might a Strict Liability system make manufacturers do more? Under negligence, its hard for the plaintiff to prove that the manufacturer was actually negligent & was therefore liable for the harm. * Why might a Negligence system make manufacturers create safer products? If the manufacturer is proven to be negligent, the jury will often give the plaintiff a much higher award in punitive & even compensatory damages. The book calls strict liability a cold tort and negligence theory a hot tort when taken to a jury, tort cases involving negligence appeal to a greater sense of justice than those involving strict liability. Often, plaintiffs will bring suit under both theories, since a finding of negligence will augment any awarded damages. * Rationales for Strict Liability: - Puts liability on best cost bearer/cost shifter. - Ease of proof. - Deterrence: Seller subject to strict liability will take additional precautionary steps. - Puts liability on best cost avoider. - Fairness: seller profits from sale of product; should bear responsibility for loss. PRODUCTS LIABILITY 2009 Matthew J. Weaver Page 14

MANUFACTURING DEFECTS Magnuson v. Kelsey-Hayes Co. page 189 * RULE: To prove a manufacturing defect, one need not prove that the product deviated from its design specifications. * This case demonstrates one of the problems with trying to place each case under a specific heading. The manufacturing problem was more of a design defect than a manufacturing defect. However, this didnt matter to the Magnuson court. The material used to create the defendants product created an unreasonable risk that resulted in injury to the plaintiff, and therefore created a liability. * RST 3d of Torts: Products Liability, section 2(a): A product has a manufacturing defect if it departs from its intended design even though all possible care was exercised in the preparation and marketing of the product. (not used in this case because RST3d wasnt written yet) Pouncey v. Ford Motor Co. page 193 * RULE: A jury may infer manufacturer negligence from circumstantial evidence where there is direct evidence in the record of an actual defect in the product. * Battle of the experts (We skipped this case) Ducko v. Chrysler Motors Corp. page 199 * RULE: A malfunction by a product establishes a prima facie case of product defect. * Here, the plaintiffs expert doesnt testify that a particular aspect of the product was defective he just calls the failure of the product a transient malfunction. * Restatement 3rd of Torts: Products liability, section 3: existence of a defect may be inferred when the injury-causing event: was a kind that ordinarily occurs as a result of a product defect; and was not, in the particular case, solely the result of causes other than product defect existing at the time of sale or distribution. (Page 202) For Monday, February 16, and Wednesday, February 18, please read pages 203-264 of the text and Webster v. Blue Ship Tea Room, which is posted in the Resources area. (Statutory supplement has Indianas version of RST2d 402A in it its different than 402A, but it starts out the same) MANUFACTURING DEFECT: RST 3rd of Torts: PL section 2(A): a product has a manufacturing defect if it departs from its intended design even if though all possible care was exercised in the preparation and marketing of the product. PROOF: EXPERT TESTIMONY: Proof of defectiveness and causation often requires scientific and technical experts to explain the relevant science and engineering of product safety and accidents to a jury and the court. Thus, involving as it does the inner workings of science and technology, products liability litigation often resolves into a battle of the experts.

PRODUCTS LIABILITY 2009 Matthew J. Weaver

Page 15

Main Question: If youre dealing with an expert that is offering scientific testimony, what are you going to require of that expert? Frye v. U.S. (page 204) * Frye was a 1923 murder case where the defendant offered the results of an early polygraph test to show his innocence. * The test established by the court as to whether this kind of evidence would be allowed is whether the form of science or technology is sufficiently established to have gained general acceptance in the particular field in which it belongs. * Under this old rule, evidence must be based on principles that are generally accepted within the scientific community testimony had to be measured against external standard: a standard put forth by the scientific community. Daubert v. Merrell Dow Pharmaceuticals page 205 * RULE: An expert's opinion need not be based on techniques generally accepted in the scientific community in order to be admissible in a federal court. The subject of an expert's testimony must be "scientific knowledge," which means that it must be derived from the scientific method and be subject to scientific considerations such as testability, error rate, control standards, peer review, and general acceptance in the pertinent scientific community. * INFO: Rule 702 is not the only Federal Rule of Evidence applicable to expert scientific testimony. Rule 703 provides that expert opinions based on otherwise inadmissible hearsay are to be admitted only if the facts or data are reasonably relied on by experts in the relevant field. Rule 706 permits the court to appoint an expert of its own choosing, and Rule 403 permits the exclusion of relevant evidence if its probative value is outweighed by the danger of unfair prejudice. * CAUSATION: The aggregate effect of preceding events that bring about a tortious result; the causal connection between the actions of a tortfeasor and the injury that follows. DAUBERT FACTORS: Factors for 1. Testability evaluating 2. Error rate (lower the error rate, the more reliable) expert 3. Control standards testimony 4. Peer review and publication 5. General acceptance (this, the Frye test, is still a factor) * This was meant to be a flexible standard, but in practice, courts use these factors as a sort of checklist. Why? Instead of Fryes strict external standard (demand for good testimony is put on the scientific community), this test internalizes the standard it puts the test of methodology and the reliability of the conclusions drawn as one that the courts conduct, with an understanding of the scientific communitys perspective on the testimony. * The Daubert reliability test concerns apply outside of the scientific context to other cases requiring expert testimony (Kumho Tire).

PRODUCTS LIABILITY 2009 Matthew J. Weaver

Page 16

Expert Testimony: Federal Rules of Evidence 702 (amended): 1. Testimony must be based on sufficient facts and data 2. Testimony must be product of reliable principles and methods 3. Witness has applied the principles and methods reliably to the particular case Booth v. Black & Decker, Inc. page 210 * RULE: To constitute admissible evidence on a subject, an expert must provide the court with enough basic, objective information on the reliability of his or her investigation and the opinions based thereon. * Plaintiffs expert failed to provide the court with enough basic, objective information on the reliability of his investigation and the opinions that he based on his investigation; he merely examined the toaster oven and concluded it could have been safer. His testimony seemed wholly based on his own training and experience, and he provided the court with no "objective anchor" for his conclusions. * INFO: In its Booth decision, the court specifically noted that there was no credible evidence to show that the expert's examination method was subject to peer review, had a known or potential rate of error, could be measured "by existing standards," or was "generally accepted." In short, the expert here did not take sufficient care in supporting the credibility or reliability of the methodology he applied. Fire marshal didnt testify as an expert would the fire marshals testimony withstand a Daubert inquiry? Well, look at his methodology in light of the factors see how difficult Daubert makes things for experts with only experiential qualifications What constitutes good science? The Black & Decker expert did not show that his examination method was subject to peer review, had a known or potential rate of error, could be measured "by existing standards," or was "generally accepted." Here, the jury probably wouldnt have given much weight to the experts testimony, but this case didnt even get to that stage: the plaintiffs experts testimony was excluded. Rudd v. General Motors Corp. page 212 * RULE: An expert's systematic elimination of alternative causes for a manufacturing defect may constitute admissible circumstantial evidence of defectiveness. * INFO: As noted in the Rudd decision, the expert Edmondson provided a step-by-step and transparent account of "reasoning processes and data sources" on which he relied, as well as "the physical indicia he associates with each possible alternative cause, and his reasons for excluding each of the alternative causes." By fully revealing the basis of his opinions, Edmondson's testimony thus supplied General Motors (D) with a fair basis to challenge his opinions by cross-examination and the presentation of contrary evidence, the basic tool of the adversary process. Webster v. Blue Ship Tea Room (Oncourse PDF) Plaintiff alleges breach of implied warranty of merchantability of food served by defendant restaurant

PRODUCTS LIABILITY 2009 Matthew J. Weaver

Page 17

(the suit was brought under UCC 2-314(1) & (2)(c) and UCC 2-316(3)(b) see the bottom of page 2 for a summary of these UCC sections) Procedural History: trial judge refused to allow defendants motion to direct the verdict in its favor after the jury found in favor of the plaintiff Facts: Plaintiff ordered a cup of fish chowder. A fish bone was in the chowder and she accidentally swallowed it. She had to have two esophagoscopies, and the fish bone was found and removed upon the second esophagoscopy (see page 2, column 1 for more details as to how she ate the soup). Issue: When a chef leaves a fish bone in regular fish chowder, does this constitute a breach of the implied warranty of merchantability? The court says no The court looks into the history of fish chowder and takes into account the fact that the plaintiff was a native New Englander eating in a quaint restaurant, someone who should be acquainted with fish chowder. The court distinguishes the stray bone from the fish from foreign substances in other food products where the fitness or merchantability of the product would be affected, as the bone did not taint the dish as to make it unwholesome In class: the impossibility of achieving perfection is no defense to a charge of defective product liability (or whatever) In class: the product met her expectations Reasonable expectation may vary by region

Kolarik v. Cory International Corporation page 214 * Cory Intl was just an importer * Cory Intl relied on its Spanish suppliers to control the quality of the olives * RULE: The negligence of a preparer or processor of food should be assessed according to the consumer's reasonable expectations about the food as it is served. * The court says that a question of fact therefore exists on whether Cory (D) adequately warned Kolarik (P) about the possible presence of pits in the olives. * INFO: The majority rule favoring the "consumer expectations" test shifts a substantial burden to potential defendants to warn consumers of potential risks. Whereas the older "foreign-natural" test clearly would have absolved defendants in situations such as Cory's (D) here, the "consumer expectations" test makes the analysis much more fact-sensitive so much so that summary judgment on such claims is now less appropriate than it was before. * My jar of olives says: Olives may occasionally contain a pit fragment due to machine processing Shoshone Coca-Cola Bottling Co. v. Dolinski page 220 * RULE: When a plaintiff produces evidence that his injury was caused by a defect in a product that was present at the time the product left the hands of the manufacturer, it is up to the manufacturer to offer evidence of tampering with the product. * INFO: In most cases, as in this one, direct evidence of tampering or the absence of tampering is not available. The jury must infer that tampering did or did not occur based on the evidence that is available. Foreign matter is more likely to have found its way into PRODUCTS LIABILITY 2009 Matthew J. Weaver Page 18

a sealed container before leaving the factory than after as a rule, and the defendant's task of showing tampering is not an easy one. * When a manufacturer of an allegedly defective product asserts that its product was subsequently tampered with, and that this tampering caused the harm, does the plaintiff have to prove that no tampering occurred or vice versa? The court says its up to the manufacturer to offer evidence of subsequent tampering with the product: When a plaintiff introduces evidence that his injury was caused by a defect in a product that was present at the time the product left the hands of the manufacturer, it is up to the manufacturer to offer evidence of tampering with the product. Once it is decided that enough evidence is present to trace [the cause of the harm] to the defendant, that same evidence is sufficient to allow the jury to find an absence of tampering bottom of page 221 * Think about comparative negligence the burden to show negligence on the part of the plaintiff is on the defendant! DESIGN DEFECTS Matthews v. Lawnlite Co. page 227 * When is a product defective? * RULE: When a product's design conceals an inherently dangerous mechanism, the product's design should incorporate a protective housing for the mechanism. * Negligence theory case (See RST 398 page 227) RST 398: A manufacturer of chattel made under a plan or design which makes it dangerous for the uses for which it is manufactured is subject to liability to others whom he should expect to use the chattel lawfully or to be in the vicinity of its probable use, for bodily harm caused by his failure to exercise reasonable care in the adoption of a safe plan or design. The professor says that this is a straightforward negligence standard * INFO: The court's analysis does not discuss the feasibility of an adequate warning, although it is noted that there is no notice of the dangerous mechanism. It is unlikely that any warning would suffice, unless it is demonstrated that the mechanism was essential to the product and the incorporation of a protective housing would destroy the utility of the product. * Lawnlite was decided before RST2d 402A was drafted, which moves toward more of a strict liability standard RST2d 402A comment g: The rule stated in this section applies only where the product is, at the time it leaves the sellers hands, in a condition not contemplated by the ultimate consumer, which will be unreasonably dangerous to him this hints back to the implied warranty origins of strict liability RST2d 402A is generally a consumer expectation standard see also RST2d 402A comment i: the article must be dangerous to an extent beyond that which would be contemplated by the ordinary consumer who purchases it, with the ordinary knowledge common to the community as to its characteristics McCormack v. Hankscraft Co. page 228 * When is a product defective? PRODUCTS LIABILITY 2009 Matthew J. Weaver Page 19

* RULE: A manufacturer will be liable for reasonable reliance on instructions warranting safety features. A manufacturer may be held liable for a design defect when it advertises that its product is safe without disclosing the existence of a dangerous condition which is not readily discoverable by the user. When the defect can be easily remedied and the danger from normal use is readily foreseeable, the design must be deemed defective. * Sometimes it is difficult to prove that there is a reasonably feasible alternative design * Hand formula: B<PLN this somehow figures into the reasonableness of the design * INFO: Many cases in this area are resolved on a theory of misrepresentation or on a theory that the instructions or representations create a trap for the unwary. Even obvious dangers may be deemed latent defects if instructions are calculated to have the effect of masking or unduly minimizing them. Even if the defect could not be guarded against, failure to adequately warn of the danger or to instruct in the proper use will render the design unnecessarily dangerous. * The problem with strict liability of products has been one of limitation. No one wants absolute liability where all the article has to do is to cause injury. To impose liability there has to be something about the article which makes it dangerously defective without regard to whether the manufacturer was or was not at fault for such condition. A test for unreasonable danger is therefore vital page 232 Donegal Mutual Insurance v. White Consolidated Industries, Inc. page 234 * When is a product defective? * RULE: A stove is more dangerous than an ordinary consumer would expect if it causes a fire that destroys a home after the homeowner merely sets the stove to the self-clean mode. * Consumer-expectation test for unreasonable danger: Is the product "more dangerous than an ordinary consumer would expect"? Strict liability theory! * INFO: Even if other evidence at trial undermined or even contradicted Susan Nearon's (P) testimony about setting the stove to the self-clean mode, her testimony was sufficient, by itself, to sustain a verdict for plaintiffs. Under the consumer-expectations test, a stove that causes such catastrophic damage is clearly "more dangerous than an ordinary consumer would expect." Vincer v. Esther Williams All-Aluminum Swimming Pool Co. page 234 * RULE: Whether a product contains an unreasonably dangerous defect depends upon the reasonable expectations of the ordinary consumer concerning the characteristics of this type of product. * Negligence theory: this court looks to RST 402A: Even if a product is defective, it must be shown to be unreasonably dangerous to the user or the consumer. RST2d 402A comment i defines an unreasonably dangerous product as a product that is dangerous to an extent beyond that which would be contemplated by the ordinary consumer * In this case, the danger was open, obvious, broadly known and broadly appreciated kids are known to be good climbers who could climb into a swimming pool

PRODUCTS LIABILITY 2009 Matthew J. Weaver

Page 20

Nichols v. Union Underwear Co. page 242 * RULE: Although the danger of a product falls within the consumer's reasonable expectation, the product may still be unreasonably dangerous. Consumer expectation is merely one factor in determining whether a product is unreasonably dangerous. A product does not become insulated from liability merely because its danger is patent rather than latent. * Concurrence: Whether a product contains a design defect must be determined using a social utility test, that is, whether the costs of a safer design are outweighed by the benefit of the increased safety. * INFO: This majority opinion and the concurrence point out a significant issue in product design cases. This is the applicability of the social interest test. Several jurisdictions have adopted this balancing test as the definitive statement of liability. The consumer expectation test can be viewed as a restriction on liability while the social utility test takes a broader view. * RULE: Although the danger of a product falls within the consumer's reasonable expectation, the product may still be unreasonably dangerous (the risk-utility test). * B < PL D * ^^^Basically the Hand test, except w/ product is defective substituted for manufacturer is negligent (Is risk-utility simply negligence in disguise?) * The court applies a test that is not much different from the test applied in the Lawnlite and McCormick cases. * Concurrence: If youre in a situation where the burden of making some change is less than the probability of the harm multiplied by the severity of the harm, then the product is defective. This test does not suggest an alternative design it is just a cost-benefit analysis. * Things a manufacturer has to take into account in its cost-benefit analysis: think about them (any change in the product potentially may have spiraling effects product might become less desirable due to the changes) considerations must be within the competence of the court to decide * Indiana Code 34-20-2-2: [I]n an action based on an alleged design defect in the product . . ., the party making the claim must establish that the manufacturer or seller failed to exercise reasonable care under the circumstances in designing the product * So, in Indiana and many other jurisdictions, defective design is negligence, even if courts call what theyre doing strict liability in some jurisdictions * But there is often a meaningful difference: (page 252) The Wade-Keeton Test: Knowledge is imputed on the manufacturer: You will find for the Plaintiff if you are satisfied that [the defect created such a danger] than an ordinary prudent company . . ., being fully aware of the risk, would not have put [the product] on the market. Nichols v. Union Underwear jury instruction. (This is a potential way from differentiating strict liability from negligence) RST 3d: PL 2(b): A product is defective in design when the foreseeable risks of harm posed by the product could have been reduced or avoided by the adoption of a reasonable PRODUCTS LIABILITY 2009 Matthew J. Weaver Page 21

alternative design and the omission of the alternative design renders the product not reasonably safe." Sperry-New Holland v. Prestage page 245 * RULE: Courts should apply a risk-utility analysis to determine product defectiveness rather than a consumer expectations analysis. In a risk-utility analysis, a product is unreasonably dangerous, and therefore defective, if a reasonable person would conclude that the danger-in-fact, whether foreseeable or not, outweighs the utility of the product. Whether a plaintiff appreciates the danger of a product is irrelevant to this analysis. Older cases may have applied a consumer expectations analysis, but the national trend now is to focus upon the utility and safety of a product in view of its intended function. A risk-utility analysis best protects both the manufacturer and the consumer. It requires the manufacturer to make its product reasonably safe regardless of whether the consumer is aware of the product's dangerousness, while at the same time taking into account an ordinary person's ability to avoid the danger. * INFO: A risk-utility approach involves balancing the probability and seriousness of harm against the costs of taking precautions. It therefore provides an incentive to manufacturers to design their hazardous products in such a way as to minimize harm to their users. Factors to be considered when employing a risk-utility test include the availability of alternative designs, the cost and feasibility of adopting alternative designs, and the frequency or infrequency of injury resulting from the design. Cronin v. J.B.E. Olson Corp. page 258 * RULE: When a plaintiff establishes that a product contained a defect that proximately caused his injuries, he need not show that the product was furthermore unreasonably dangerous to the user or consumer. * INFO: Most cases and commentators are not in accord with the Cronin approach to strict liability. The term "defect" is viewed as vague in the absence of a determination of whether it is unreasonably dangerous. The reason the law attaches liability to the seller is that it has subjected an unsuspecting member of the public to potential injury, and the term "defect" does not make this plain by itself. Soule v. General Motors Corp. page 264 * RULE: Consumer expectations is not an appropriate defectiveness standard for complex products. * Two distinct tests exist for gauging design defectiveness: (1) whether the product fails to perform as safely as a reasonable consumer would expect when using the product in a reasonably foreseeable way; or (2) whether the dangers created by the product outweigh the benefits of the challenged design. In a complex product, it is difficult to envision what a reasonable consumer could expect in terms of safety, as the product's complexity forecloses a layman from having ordinary expectations. The second test, which requires expert testimony regarding the risks and benefits of the products, is much more appropriate in cases involving complex products. Therefore, the consumer expectations test should not be used in cases involving complex products.

PRODUCTS LIABILITY 2009 Matthew J. Weaver

Page 22

* INFO: The analysis of the Court here is generally accepted in those jurisdictions having ruled on the issue. Consumers cannot have expectations outside of their common experience. Technical products are usually not part of most people's common experience. * A lot of the products we use these days are so complex they must be magic. We dont always have the ability to form reasonable expectations about every product we use because we dont always know how the product works. Design defect Consumer Expectations Difficulties w/ consumer expectations test: 1) Known or open and obvious dangers 2) Basis for expectations Barker v. Lull Engineering Co. page 261 * INFO: Some courts have required that a product be used in an intended manner. The test employed here is somewhat more plaintiff-friendly. Under the standard here, a consumer can misuse a product from a manufacturer's point of view but still recover if the use was foreseeable. * RULE: Products are considered defectively designed if they fail to perform as safely as an ordinary consumer would expect if used in a reasonably foreseeable manner. * There were reasonable alternative designs that the manufacturer could have implemented that would have avoided the danger * The court says that there are situations in which the consumer expectations test works well, and in situations where it doesnt work well, then the court should turn to the riskutility test * Defendant has to show the reasonableness of its choices under the Risk-Utility Analysis * Problem with consumer expectations test = manufacturers can just make the dangers of their products obvious * Problem with consumer expectations test = how do you determine what the consumers expectations are? * RST3d: PL 2(b) the language here sounds a lot like negligence * Design Defect Risk-Utility: California: Shift burden to D to prove that the product is not defective Barker v. Lull Engineering Company * Design Defect Combining Consumer Expectations and Risk-Utility: California: Use consumer expectations for products for which consumers are capable of forming reasonable expectations; otherwise, use risk-utility * When a complex product does something that was really weird (ex: steering wheel pops off while you are driving or if your accelerator gets stuck), then riskutility can be applied Potter v. Pneumatic Tool Co.

PRODUCTS LIABILITY 2009 Matthew J. Weaver

Page 23

* RULE: Plaintiffs do not need to present evidence of feasible alternative designs to establish a prima facie case of design defect. * In Connecticut: integrate risk-utility into consumer expectations: Provide jury with evidence about risks and utility & ask if a reasonable consumer would view the product as unreasonably dangerous in light of the risk-utility evidence * This is not actually a consumer expectations test (here, Connecticut is trying to integrate risk-utility into consumer expectations comment g of RST3d tries to do the opposite) * INFO: Sometimes, the product's defectiveness will be so obvious to a lay jury that risk and utility factors do not need to be considered. Most of the time, however, the defective design is more subtle or the product is less familiar to the average juror and the Potter court's modified consumer expectations test provides a way (i.e., balancing risks and utilities) for consumers to define what their "reasonable expectations" are for a product's safety. * Defect cases: In this area of the law, each jurisdiction has its own rules courts arent simply adopting RST3d & there isnt much stability within the jurisdictions * RST3d: PL 2, comment g: Consumer expectations are not an independent ground for recovery but rather are part if the risk-utility analysis * RST3d takes the position that Ps should not be able to pursue multiple theories of design defect because this will possibly confuse the jury Denny v. Ford * New York: design defect = risk-utility test; implied warranty of merchantability = expectations standard * The trend in the law, however, is toward there being a single standard for product liability (this makes it easy on the jury b/c they dont have to run a single set of facts through 3 different tests) * RULE: It is possible to be liable for a breach of implied warranty even though a claim of strict products liability has not been satisfactorily established. * INFO: The dissent reasonably fears that the introduction of contract law analysis into tort law claims will destabilize the arena of personal injury. The problem is that the consumer expectation analysis supplied by the majority is a standard of proof that is often easier to prove than the risk/utility approach of strict liability. Plaintiffs will be tempted to cry that their expectations were not met, whether or not they sincerely had such expectations at the time of purchase. Knitz v. Minister Machine Defective product Ps expert introduces known alternative designs D wins * RULE: In a products liability case based upon defective design, the plaintiff has the burden of proving the availability of a cost-effective, alternative design for the product. * INFO: In determining the availability of a feasible alternative design, the trier of fact must focus not only on the cost of the alternative design, but also on the new design's impact on the overall operation and utility of the product. On the evidentiary issue raised PRODUCTS LIABILITY 2009 Matthew J. Weaver Page 24

by the court, it is clear that compliance with applicable standards is not conclusive on the issues of tort liability. * Just because a product complies w/ govt safety standards does not mean its not defective Design Defect: Delegation where a manufacturer places into the channels of trade a finished product which can be put to use and which should be provided with safety devices because without such it creates an unreasonable risk of harm, and where such safety devices can feasibly be installed by the manufacturer, the fact that he expects that someone else will install such devices should not immunize him Design Defect: Delegation (these questions are often difficult to answer) * Is the product supplied with a finished (usable) form? * Does the product in that form, and without safety devices, present an unreasonable risk of harm? * Is it feasible for the manufacturer to supply safety devices? Expert Testimony: Federal Rules of Evidence 702 (amended) 1. Testimony must be based on sufficient facts and data; 2. Testimony must be product of reliable principles and methods; 3. Witness has applied the principles and methods reliably to the particular case. Expert Testimony: Daubert Factors 1. Testability; 2. Error rate; 3. Control standards; 4. Peer review and publication; 5. General acceptance Calhoun v. Yamaha Motor Corp. page 293 * RULE: In the absence of an abuse of discretion, the admissibility of expert testimony is entirely discretionary with the trial court. * Ideal witness: person who had experience with the design of the jet ski very difficult to get someone in the field to serve as the plaintiffs expert * Lawyers responsibility, ultimately to explain to the experts what to do in order to get their opinions in front of the jury and to have the experts present their testimony in an acceptable way * INFO: As noted in the Calhoun decision, an expert may be generally qualified but may lack qualifications to testify outside his area of expertise. Hence, a particular expert may be permitted to give an opinion as to certain aspects of the case, but not necessarily as to every aspect of the case.

PRODUCTS LIABILITY 2009 Matthew J. Weaver

Page 25

3 tests: Consumer expectations Unreasonable danger not within the contemplation of the consumer Does not allow for obvious dangers Risk-utility Essentially a negligence test But imposed retrospectively even if the dangers are not knowable So maybe this is strict liability Hybrid Consumer expectations where dangers are not open and obvious and where consumers can form an expectation otherwise, use risk-utility WARNING DEFECTS: Lewis v. Sea Ray Boats page 302 * RULE: In a products liability "failure to warn" case, the jury should be instructed that "adequate warning" means the warning was designed to reasonably catch the consumer's attention, sufficiently comprehensible, and of sufficient intensity justified by magnitude of the risk. * INFO: As clearly indicated in the Lewis decision, a party is entitled to have jury instructions on all their theories of the case that are supported by the evidence, and general abstract or stock instructions on the law are insufficient if a proper request for a specific instruction on an important point has been duly proffered to the court. Warning Defect: elements of an adequate warning: 1. Conspicuousness: - Warning must be printed in type of sufficient size and color to draw attention; - Warning must be headed by appropriate signal words 2. Scope: - Warning must adequately inform users of nature and extent of danger 3. Intensity: - Warning must be of intensity justified by magnitude of risk * Warning: we want manufacturers to warn about the more likely dangers we want them to avoid over warning because users will skip over a long and detailed warning section Olson v. Prosoco page 313 * RULE: Jury instructions regarding a failure-to-warn claim for damages must be submitted under a theory of negligence rather than a theory of strict liability. * Is it appropriate to submit both strict liability and negligence standards to the jury? No! The court says that any difference between the two theories is illusory and submitting both confuses the jury. In the context of a warning defect, you only have to submit negligence, the court says, because what you are really doing is assessing the reasonableness of the manufacturers decision to warn in the way they did. PRODUCTS LIABILITY 2009 Matthew J. Weaver Page 26

* INFO: If the negligence-only doctrine is employed, establishing liability in a failure-towarn case requires a showing that the manufacturer knew or should have known of the danger, in light of prevailing best scientific knowledge, yet did not provide adequate warning to users or consumers. The manufacturer is held to the standard of care of an expert in its field. The new Restatement of Torts, on the other hand, defines a product as "defective" (thereby rendering its manufacturer strictly liable) when the foreseeable risks of harm posed by a product could have been avoided by a warning, and omission of the warning makes the product not reasonably safe. See 2(c). Why havent the courts taken the other approach??? Warning Defect: Tests 1. Impute present knowledge of products risks to manufacturer; ask whether warning is reasonable in light of those risks, even if risks were unknown and unknowable at the time the product was sold. (This is the Wade-Keeton test many courts reject this test) 2. Consumer expectations. (At the heart of 402A, according to its comments) 3. Negligence. Indiana Code 34-20-2-2: [I]n an action based on an alleged design defect in the product or based on an alleged failure to provide adequate warnings or instructions, the party making the claim must establish that the manufacturer or seller failed to exercise reasonable care under the circumstances in designing the product or in providing the warnings or instructions. Meyerhoff v. Michelin Tire Corp. page 317 conspicuousness of a warning * RULE: To support a finding of a failure to warn, expert testimony regarding alternative warnings must establish feasibility, adequacy, and effectiveness. * Tire explodes, killing a man trying to inflate it Plaintiff argues that the warning on the tire was not conspicuous the warning provided was embossed on the tire, but it was not painted in (the yellow warning theory) * The court enters a judgment for the defendant the court says that an adequate warning will adequately inform consumers of the dangers posed by the product as a whole and a particular focus on a particular danger may misrepresent the totality of the danger posed by the product as it focuses on a single danger posed by the product and possibly downplays the other dangers * To prevail, the plaintiffs expert has to present a better warning and establish the feasibility, adequacy, and effectiveness of this better warning also, this expert testimony must pass the Daubert test. * INFO: Other issues regarding the adequacy of warnings include the conspicuousness and location of the warning. Warnings must be of a sufficient size and color. They should also be included on the product itself, if feasible, and use signal words such as "Warning" or "Danger." * We want manufacturers to provide a warning, even if the consumer is not going to read the warning. This could create a kind of comparative negligence in the case that a court finds the warning to not be conspicuous enough PRODUCTS LIABILITY 2009 Matthew J. Weaver Page 27

Pictorial Warnings: great photos starting on page 323 * The pictorial warning has convey the right message in the right context, to the targeted individual Higgins v. DuPont page 331 delegation of warning obligation * RULE: There is no duty on product suppliers to warn ultimate users of product-related hazards in products supplied in bulk to a knowledgeable user. * Sophisticated user defense: the consumer supplied with the product knew of the dangers of the product because they were experts in the * Bulk supplier defense: the bulk supplier did not have an effective way of communicating with the end user, because they only sold a component of the final product, which was assembled by the consumer and then finally sold to the end user * It is the duty of the products manufacturer, who manufactured the product that was supplied to the individual who suffered harm, to provide an adequate warning to the end user * INFO: Not all courts adopt the reasoning in the present case. Some courts find that a supplier's obligations to provide warnings to the ultimate user is a question of fact, with the risk posed by the product balanced against the feasibility of the warning. Higgins (P), in the present case, could have argued that no steps were taken to ensure that Dupont (D) was promulgating an effective warning, undercutting the "reasonable" nature of moving parties' reliance. * Trained professionals are sophisticated users Warning Defect: Delegation RST (rd) of Torts: Products Liability 5: No liability for component part suppliers unless: (a) The component is defective in itself, as defined in 2, and the defect caused the harm; OR (b) 1. The seller or other distributor of the component substantially participated in the design of the product 2. The integration of the component causes the product to be defective as defined in 2; AND 3. The defect in the product causes the harm. For Monday, March 9, and Wednesday, March 11, please read pages 340-49 and 398436. It's law firm #2's turn this week. Prescription Drugs

PRODUCTS LIABILITY 2009 Matthew J. Weaver

Page 28

Lavigne v. Wyeth failure to warn case Supreme Court allowed state failure to warn case to go forward we will talk about this case later in the semester (involvement of FDA does not preempt state prescription drug failure to warn cases) Under the common law, to whom is the duty to warn owed? Learned intermediary doctrine pharmaceutical companies owe a duty to warn to the prescribing physician, not the patient to whom the drug is prescribed this is a variation of the sophisticated user concept this doctrine says that pharmaceutical companies are not in a position to communicate a warning effectively to the patient and it also says that because of the doctors knowledge of the patient, the doctor is in a position to tailor whatever warnings and instructions the doctor is going to provide to the particular needs of the patient most states recognize at least some exceptions to this doctrine for instance, in the area of mass immunizations here, the immunization is administered by a nurse and the nurse is not in a position to provide those warnings and therefore the warnings should come from the manufacturers of those immunizations another exception is in the case of oral contraceptives here, its a situation where the courts made the decision that the discussion between the doctor and patient is usually different its an area in which the patient is generally more involved in choosing what kind of medication to take and the doctor plays more of a background role also, oral contraceptives are usually prescribed on a monthly basis another exception is in the area of direct marketing to the consumer if you dont talk about the benefits of the drug, then you dont have to talk about the dangers of the drug (remember the Claritin commercial from the 1990s that we saw in class) this is because direct marketing changes the nature of demand for prescription drugs in an idealized world, before direct marketing, the doctor would listen to the patients symptoms and then tell them about a drug but now, with direct marketing, patients now go and demand drugs commercials need to say what possible side effects a medicine will have in their advertising, because, if a commercial just tells the positives and not the negatives then maybe the patient will ignore their doctor & doctor shop see also Perez v. Wyeth (page 348), which is real mean to the learned intermediary doctrine but didnt catch on, probably b/c Perez is a NJ case & the NJ supreme court was known at the time for taking things to the extreme Open and Obvious Dangers If you go to the zoo and put your hand through the lions cage, and the lion bites your hand off, you do not have an action against the zoo Judge Posner said this is he wrong? General rule is that the manufacturer does not have to design away open and obvious dangers Autonomy concerns spaced out, sorry Campo Rule the vast majority of jurisdictions do not follow this rule Obvious Dangers: Failure to Warn

PRODUCTS LIABILITY 2009 Matthew J. Weaver

Page 29

Belling v. Haughs Pools page 402 * RULE: In a situation where there is an open and obvious danger, there is no duty to warn why? because the warning wont add anything a warning would have merely informed [the consumer] of risks of which he was already aware Inherent Dangers Inherent Dangers: Restatement (2d) 402A, comment i: [god damn it] Good whiskey is not unreasonably dangerous merely because it will make some people drunk, and is especially dangerous to alcoholics Good tobacco is not unreasonably dangerous merely because the effects of smoking may be harmful * Tobacco spiked w/ marijuana would be bad Bruner v. Anheuser-Busch page 409 * RULE: Under the Restatement (Second) of Torts 402A, alcoholic beverages are not an "unreasonably dangerous" product. * RST2d 402A, comment i * INFO: As noted in the Bruner decision, traditionally courts have recognized that although there are dangers involved in the use of alcoholic beverages, because of the common knowledge of these dangers, the product is not considered unreasonably unsafe. McMahon v. Bunn-O-Matic page 413 * RULE: Failure of a manufacturer of a coffeemaker to inform consumers that the maker brews coffee which is extremely hot does not constitute a "failure to warn" nor make the coffee a defective product unfit for human consumption. * INFO: As made clear in the McMahon decision, most courts do not require vendors to give warnings in extreme detail, but expect consumers to a great extent "to educate themselves about the hazards of daily life of matches, knives, and kitchen ranges, of bones in fish, and of hot beverages by general reading and experience, knowledge they can acquire before they enter a mini mart to buy coffee for a journey." Liebeck v.McDonalds page 417 (note) * The famous McDonalds case * Better developed factual record in this case than in Bunn-O-Matic * McDonalds knew there was a problem w/ their coffee being too hot they didnt want to fix the temperature b/c the high temperature masked the nasty taste * The court found that McDonalds served the coffee at a temperature that was not consumable Inherent Dangers: Restatement (3d) 2, comment e: * Alternative reasonable design is almost always required. * Product category liability possible where the extremely high risk of danger posed by its use or consumption so substantially outweighs its negligible social utility that no rational, reasonable person, fully aware of relevant facts, would be able to use, or allow children to use the product PRODUCTS LIABILITY 2009 Matthew J. Weaver Page 30

Inherent Dangers: lawn darts example under comment e, this is a situation where it would go to the jury lawn darts = a situation where there is such negligible social value of the product and such an extremely high risk of danger that lawn darts have been banned in the U.S.A. March 23, 2009 Tobacco Tobacco: one of the few products that courts refused to impose liability, in the past Inherent Dangers: Restatement (2d) 402A, comment i: lists tobacco & alcohol as two products where dangers cant be designed out is the product defective if the dangers cant be designed out? No this is what comment i says Hearn v. R.J. Reynolds Tobacco page 419 * RULE: Comment i of The Restatement (Second) of Torts 402A does not bar all smokers' products liability suits. * The court refuses to go along with the comment i doctrine * The court says that this isnt good tobacco because there are additives in the cigarettes to make the cigarette more addictive * This is basically a risk-utility reasonable alternative design analysis the product could be made safer by removing the additives * Viewed from a consumer expectations perspective, this can be seen as a consumer expectations case because cigarette customers didnt know that the tobacco companies were manipulating the product to make it more addictive * The court basically says that we dont need product category liability in these types of cases Products ought to bear the costs of the injuries associated with them State of the Art State of the Art evidence tests the idea that negligence is not required in product liability claims Bruce v. Martin-Marietta page 462 * RULE: State-of-the-art evidence is relevant to a strict liability claim. * Consumer expectations jurisdiction * Plane crash happens in 1970 plane was made in 1952 Plaintiff says that the seats could have been made safer Defendant (correctly) says that the plane was considered safe in 1952 summary judgment for the Defendant * Consumers understand that a Model T is less safe than a 2009 car. Does the consumer have an expectation about how the seats in a plane will react when the plane crashes?

PRODUCTS LIABILITY 2009 Matthew J. Weaver

Page 31

* The court frames its analysis in terms of what reasonable consumers will expect is that what is going on here? Well, the court approved the grant of a summary judgment & usually in consumer expectation cases the jury is the stand-in for the consumer, who hears the evidence and determines whether there was a reasonable expectation * INFO: This case seems to represent the majority view on the issue of whether state-ofthe-art evidence is admissible in a case based upon a claim in strict liability. The intermediate appellate courts in Illinois have taken a contrary position, and have held that state of the art evidence is not relevant to a strict liability claim. This view is based upon the principles of the restatement that a seller may be liable in strict liability even though "the seller has exercised all possible care in the preparation and sale of his product." See Restatement (Second) of Torts, 402(A)(1)(a). Boatland v. Bailey page 463 * Can the Plaintiff offer State of the Art evidence when suggesting that a product was defective because there was a reasonable alternative design? * The court says that the evidence offered by the P was beyond the state of the art this is because there was a patent pending on the alternative design the design was not industry standard the court says that industry standards are different from the state of the art state of the art means state of industry knowledge * RULE: Evidence regarding state of the art may be introduced in a products liability action based on defective design. * The court DID let the Ps evidence be submitted to the jury though no suggestion in the courts opinion on what the jury should be told when presented with evidence such as this. * INFO: The major issue in "state-of-the-art" analysis is exactly what the phase refers to. In a narrow sense, it can mean industry custom. This is usually the defendant's argument. In a broader sense, it can encompass the availability of all of modern technology. Jurisdictions vary as to which approach is correct. The role of state of the art evidence: Under Wade-Keeton, analysis of state of the art evidence is usually pretty straightforward. How do you analyze this kind of evidence under the consumer expectations approach? (it can be done) Beshada v. Johns-Manville page 468 * Alert: this is a 1980s New Jersey case the NJ SC was pretty out there in the 80s * This is an asbestos case * Failure to warn case * Defendants put forth a state of the art defense that the knowledge necessary to provide the warning to the consumer did not exist at the time * RULE: Defendants in a products liability case based on strict liability for failure to warn may not raise a state-of-the-art defense. * NJ SC says that Ds cant use this type of defense in strict liability cases * NJ SC says that the state of the art defense is a negligence defense D is basically saying that they werent negligent because they didnt know about the harm their product would cause NJ SC says who cares this is strict liability this is because we want to encourage manufacturers to create safer products and manufacturers are in a better PRODUCTS LIABILITY 2009 Matthew J. Weaver Page 32

position to deal with these costs of innovation than consumers but the court recognizes that shifting these costs to the manufacturer may have its problems * INFO: The Beshada court was careful to circumscribe its holding by limiting it to failure-to-warn cases only. It specifically declined to decide whether a state-of-the-art defense would be appropriate in safety device cases. Nevertheless, it noted that there were strong conceptual similarities between warning and safety device cases, especially since, in both types of cases, the defendant's knowledge of the danger was irrelevant. * In the end: NJ SC applied Wade-Keeton in failure to warn cases, state of the art defenses are not allowed * BUT look @ the next case Feldman v. Lederle Labs page 475 * This is a NJ SC case * RULE: Strict liability will be applied to drug manufacturers. While some sorts of health-related activities are exempt from strict liability rules based on public policy concerns, the manufacture and distribution of drugs is more like the standard product distribution system. It also cannot be said that strict liability should not be applied because drugs are unavoidably dangerous. Some drugs may be, but not all are. * 2 years after Beshada (a short amount of time), the NJ SC says that in failure to warn cases, state of the art defenses are allowed * However, the NJ SC says, We do no overrule Beshada, but restrict Beshada to the circumstances giving rise to its holding. * RST 2d 402A, comment k immunizes from strict liability the manufacturers of some products, including certain drugs, that are unavoidably unsafe uses as an example the rabies vaccine, which often leads to serious injury to the end user * The NJ SC, instead of saying that RST 2d 402A, comment k applies in all product liability cases involving drug manufacturers, alternatively says that they will decide whether comment k applies on a case-by-case basis why? because the underlying rationale for comment k does not seem to apply for drugs that are not lifesaving (ex: Rogaine, beauty products, etc.) * What cases like Feldman do is bring courts face to face w/ the idea that sometimes liability will be imposed on a manufacturer who could not know the risk of harm * INFO: It appears that strict liability is something of a misnomer in describing a defective warning cause of action. Strict liability supposedly has no concern for a manufacturer's conduct, and this is the case for manufacturing defects. Design defects and defective warnings do focus on the level of care by a defendant. This is not strict liability at all, but rather negligence analysis. Brown v. Superior Court page 481 (just a note) * California DES case * CA Court says that the only standard that would apply in failure to warn of a risk of harm posed by a drug cases was negligence Anderson v. Owens-Corning page 483

PRODUCTS LIABILITY 2009 Matthew J. Weaver

Page 33

* RULE: In a strict products liability action for lack of adequate warnings, the defendant may show that the particular risk was not knowable at the time of the manufacture and distribution of the product. * The CA SC held that a defendant in a strict products liability action based upon an alleged failure to warn of a risk of harm may present evidence of the state of the art, i.e., evidence that the particular risk was neither known nor knowable by the application of scientific knowledge available at the time of manufacture and/or distribution. * Its up to the D to prove that the D didnt know and couldnt have known about the risk of harm this is what differentiates the rule from negligence, where the plaintiff must prove the defendant was negligent * The dissent/concurrence says that the court should just admit that what they are doing here is applying a negligence standard * INFO: This decision is in accord with the majority of jurisdictions who have ruled on the issue. Prior to the 1990s, many cases came to the opposite conclusion *** General Rule in the U.S. = State of the art evidence is relevant, even in non-strict liability cases Post-Sale Duties Ostendorf v. Clark Equipment Co. page 499 * RULE: A seller has no common law duty to retrofit an existing product that was not defective at the time it was sold. * The court says that there are 2 situations in which a retrofitting argument could possibly be made: 1 = if there is a latent defect at the time the product is sold if there is a latent defect that causes harm, then plaintiff can bring a claim under a negligence or strict liability; 2 = if the product was considered reasonably safe when it was sold, but times have changed and there is now a way to retrofit the product to reduce or eliminate the now-regarded risk of harm * HOWEVER the court refuses to recognize a duty to retrofit because this would actually create a situation where manufacturers are discouraged from developing safer products, since when a manufacturer improves safety features on an existing product, if a duty to retrofit were recognized, the manufacturer would have to retrofit all the nowoutdated products it previously sold * INFO: As noted in the Ostendorf decision, there is in reality no need for a court to establish a duty on the part of a manufacturer to retrofit its product because of already existing negligence and product liability doctrines. A party injured by a product can bring suit for that injury under three different theories: (1) breach of warranty under the Uniform Commercial Code; (2) negligence; or (3) strict liability in tort. * The court refused to adopt RST 3rd 11 Greiner v. Volkswagenwerk Aktiengesellschaft * RULE: When the presence of a warning could not have prevented an accident, its absence cannot be a cause in fact of that accident. * INFO: A product may be properly manufactured and well designed, yet be found defective because of lack of an adequate warning. From the facts of the Greiner cases, the PRODUCTS LIABILITY 2009 Matthew J. Weaver Page 34

most obvious defect would seem to be the Volkswagen's propensity to overturn on sharp steering maneuvers. However, the jury found that the car was not defectively designed. There was no issue raised of defective manufacture. Therefore, in order to prove a defect, Greiner (P) alleged that the warning was inadequate. Nissen Trampoline Co. v. Terre Haute First Nat'l Bank * RULE: In cases involving failure to warn, the presumption is that an adequate warning would have been read and heeded, but it may be rebutted by a showing that absence of the warning was not a cause of the accident. * INFO: Whether or not an injury could have been prevented by a warning must always be speculative. One can speculate that a young man wishing to have fun at the beach is unlikely to be deterred from using a contraption placed there for his use. However, the relatively small area of the trampoline compared to the area of the circle formed by the frame might make the product look dangerous. The appearance, along with the warning, might have deterred Garzolini (P) from using the device. * RST2d 402A, comment j: Where warning is give, the seller may reasonably assume that it will be read and heeded; and a product bearing such a warning, which is safe for use if it is followed, is not in defective condition, nor is it unreasonably dangerous. * Comment does not deal w/ issue of causation; it deals with the issue of defect * Imposes a burden of production its up to the defendant to come forward with evidence rebutting the presumption * Gives an incentive to the manufacturer to provide adequate warning * Even understanding that the warnings wont always be read, we want the warnings to be there to make products safer PROXIMATE CAUSE Crankshaw v. Piedmont Driving Club, Inc. * RULE: If the damages are only the imaginary or possible result of the tortious act, or other and contingent circumstances preponderate largely in causing the injurious effect, such damages are too remote to be the basis of recovery against the wrongdoer. * INFO: The rule of the Crankshaw case is simply the statement of the nature of proximate cause as contained in the Georgia statutes. It is but one of numerous statements of that concept. It involves a question not so much of whether or not the defendant's action caused the harm, but rather of whether or not the defendant had a duty to protect the plaintiff from the harm sustained. Piedmont's (D) negligence was the cause in fact of Crankshaw's (P) injury since "but for" the negligence, there would have been no injury. The question then is: Did Piedmont (D) have a duty to protect Crankshaw (P) from the consequence of its negligence toward her companion? Although it did not apply the term "foreseeability" to the facts, the court seems to think it unforeseeable that one would sustain injury in the way Crankshaw (P) did, and thus Piedmont (D) owed her no duty.

FORESEEABLE USERS

PRODUCTS LIABILITY 2009 Matthew J. Weaver

Page 35

Winnett v. Winnett * RULE: A manufacturer is not liable for injuries resulting from use by a person who is not a reasonably foreseeable user of the product causing such injuries. * The design here was reasonable, but a kid was using the product, and a kid is an unforeseen user * INFO: It cannot be questioned that a product which is reasonably safe for some users is dangerous in the hands of others. An automobile, safe in that it is free from defects endangering a competent adult driver, is unsafe with a child behind the wheel. It is nonetheless not defectively designed, and the manufacturer should not be subjected to liability for injuries resulting from such use of the product. Richelman v. Kewanee Machinery & Conveyor Co. * RULE: A manufacturer's duty to design a product that is reasonably fit for its intended use encompasses foreseeable ancillary consequences of normal use. * INFO: Both the majority and dissent indicate an almost conscious intention of the designers of the auger not to make it any safer than it was. The question could be phrased as one of policy: Should the designer be required to make the machine safe for operators with big feet; for all operators; for bystanders; for children who wander by; for children who want to play with the machine? The majority phrased its answer in terms leaving out only the last category; the dissent would perhaps protect only operators with big feet * If there would have been a reasonable design, then the kid wouldnt have been hurt * Causation or duty question: Palsgraf duty to do something Palsgraf analysis * Harm was even foreseeable to the appropriate users of the product adult users INTERVENING CAUSE Dugan v. Sears, Roebuck & Co. page 556 * Does the act of a 3rd party supersede a sellers breach of duty? To answer that question, one must first ask, Could the seller have foreseen the 3rd partys actions? * Neighbors negligence in ignoring warning to turn lawn mower off when others are near broke the chain of causation between the possibly defective lawnmower and the kids hurt eye * RULE: The negligence of another may break the chain of causation between a defective product and an injury. * The concept of superseding cause, in which an act of another, not foreseeable and probable, breaks the legal causation chain between an act and an injury this is the definition of superseding cause * INFO: Superseding cause is discussed in the Restatement (Second) Torts, 440-442. Various factors are discussed in these sections regarding exactly what can constitute superseding cause. No hard rule can be distilled in this regard. Whether or not superseding cause exists must be seen as an exercise in factor analysis. * Courts favor safe designs over good warnings so why is it proper for a defendant to rely on the hey we gave a warning defense? The person injured by the product was not the person who was given the warning it sounds like Cooper doesnt like this the plaintiff probably had a bad lawyer in this case, he says assuming that there really is a viable reasonable alternative design PRODUCTS LIABILITY 2009 Matthew J. Weaver Page 36

Anderson v. Dreis & Krump Manufacturing Corp. * When determining foreseeability, courts dont look at whether the specific sequence of events was foreseeable; the courts instead look at whether the harm was foreseeable * RULE: Only intervening acts that are not reasonably foreseeable may be considered superseding causes. * Was 1 button modification superseding cause? Court says its up to a jury to decide * The court says that the intervening act of the modification of the press was reasonably foreseeable by the manufacturer (or not reasonably unforeseeable as a matter of law), and thus the modification may not have been a superseding cause and may not have broken the causal chain this is up to a jury to decide * Dissent says that the modification of the press was the proximate cause of the plaintiffs injury * INFO: The majority noted that Comet's modification did not result in a different type of harm than otherwise would have occurred from a failure to supply guards with the press. The majority disputed the dissent's position, finding that the modification was not independent of the lack of guards. They stated that Anderson's (P) hand would not have been injured if there were guards, no matter how the press was activated. * Courts wont say that the mere fact that the customer made modification to the product renders the seller not liable Price v. Blaine Kern Artista, Inc. * Bush 41 mask case * RULE: Criminal or tortious third-party conduct only severs the chain of proximate causation when it is not reasonably foreseeable. * INFO: This decision is in general accord with the majority of cases throughout the country. The key question is always how narrowly or broadly to frame the question of foreseeability. See 448 and 449 of the Restatement of (Second) Torts for more information on superseding causes. Williams v. RCA Corp. * How was the product marketed? We dont know here but courts should look to this, though also, where is the substantial portion of the market? * RULE: An intervening criminal act does not necessarily break the causal connection between the defective product and the injury if that act is probable and foreseeable * INFO: Since the Williams court knew of only one case where a manufacturer was found liable where a criminal act intervened between the defect and the injury, the Williams facts must be very unusual. Note that the complaint involved strict tort liability, and thus under 402A of the Restatement (Second) of Torts, in order to prevail the plaintiff must show that the defective condition is unreasonably dangerous, which seems on its face unlikely when the defect simply caused a failure of radio communication rather than a direct injury such as electric shock. * RST2d 442 says that its appropriate for a court & jury to determine whether the 3rd partys act was a superseding cause In re September 11 Litigation PRODUCTS LIABILITY 2009 Matthew J. Weaver Page 37

* RULE: The acts of terrorists do not, as a matter of law, necessarily constitute a superseding cause that will defeat proximate cause in tort litigation. * INFO: In the In Re September 11 decision, the court noted that the danger that a plane could crash if unauthorized individuals invaded and took over the cockpit was the very risk that Boeing (D) should reasonably have foreseen. "Privacy" within a cockpit, said the court, means very little if the door intended to provide security is not designed to keep out potential intruders. Week of April 6 PLAINTIFF'S CONDUCT 436-49, 579-616 MISUSE Miller Bottling Company * 8 year old finds a beer bottle and throws it against a telephone pole * The court says: Goods must be fit for ordinary purposes, but not extraordinary purposes will the product stand up to slight misuse (not abuse, though)? Consumer expectations the kid wanted to break the bottle and he did! * RULE: A manufacturer's warranty of product fitness for ordinary use does not extend any guarantee that the product will withstand, in a reasonably safe manner, misuse not incident to or arising out of the product's intended use. * INFO: Although the courts are split as to whether this is an affirmative defense, the modern trend is to consider misuse as part of the plaintiff's case. In those jurisdictions where the plaintiff has this burden of proof, it is a factor pertaining to the issues of defectiveness and proximate cause. Sherne Lingerie * Alleged misuse: she was wearing the nightshirt inside out court says that its an error to have the jury even consider that the plaintiff was misusing the product its foreseeable that the consumer will wear the nightshirt inside-out its a misuse that is close to the intended use of the product we want the manufacturer to anticipate this kind of misuse * RULE: When it is "reasonably foreseeable" that a product will be used in a certain manner, such use is not a misuse of the product. * INFO: The standard that appears to be emerging is that if the use of a product is unusual or abnormal, and not related to the actual, designed purpose of the product, it will be considered misuse. Note, however, that if a product is continually misused in a dangerous, unintended manner, the use is probably reasonably foreseeable. Moran v. Faberge page 440 * RULE: The duty of a manufacturer to warn of latent dangers inherent in its product extends to all uses that are reasonably foreseeable. * Teenagers put perfume of a candle lol no flammable warning on the bottle * Isnt this an intervening cause case?

PRODUCTS LIABILITY 2009 Matthew J. Weaver

Page 38

* Is it foreseeable that someone will put perfume on a candle? No but its foreseeable that perfume is flammable this is something weird something that wouldnt normally happen * The court says that when were considering foreseeability, we shouldnt focus on the exact chain of events that led to the incident wont relieve liability unless the court says that the danger that manifested itself is foreseeable so in this case, it is foreseeable that the perfume could come into close contact with flames like trying to smoke while putting on perfume or knocking it over by an open flame the kind of danger is the same * Tortfeasors dont owe the world at large a duty they owe certain people a duty the dissenting opinion says that there isnt a duty owed to this particular plaintiff * INFO: The comments to the Restatement, in discussing manufacturer liability for unanticipated uses, states that: "The manufacturer may . . . reasonably anticipate other uses than the one for which the chattel is primarily intended. The maker of a chair, for example, may reasonably expect that someone will stand on it; and the maker of an inflammable cocktail robe may expect that it will be worn in the kitchen in close proximity to a fire." Restatement (Second) of Torts, 395, Comment K. Whats important isnt necessarily reaching the right result, its knowing how to frame your arguments, b/c the individual results of certain cases can sound peculiar Contributory Negligence Carlyle & Martin, Inc. page 579 * RULE: Contributory negligence will bar recovery in a negligence action. * Contributory negligence is a defense it is only considered in court if the product is defective * Would a reasonable person know of the risk? Yeah, b/c it is an open & obvious danger * INFO: The test is not whether the plaintiff actually knew of the danger confronting him, but whether, in the exercise of reasonable care, he should have known he was in a situation of peril. McCown v. Intl Harvester page 583 * RULE: Contributory negligence is not an available defense to strict liability actions. * Court rejects contributory negligence as a defense to actions under RST 402A * Contributory negligence shouldnt apply as a defense to strict liability actions * Recognition of consumer negligence as a defense to a 402A action would contradict the normal expectation of product safety. This is a policy rationale * The court says it relies on RST 402A, comment n, which says: Contributory negligence of the P is not a defense when such negligence consists merely in a failure to discover the defect in the product, or to guard against the possibility of its existence. * The court misread comment n, though * INFO: The majority opinion was based in part upon Comment N to 402A of the Restatement (Second) of Torts. That section provides in relevant part: "Contributory negligence of the plaintiff is not a defense when such negligence consists merely in a failure to discover the defect in the product, or to guard against the possibility of its PRODUCTS LIABILITY 2009 Matthew J. Weaver Page 39

existence . . . (However) if the user or consumer discovers the defect and is aware of the danger, and nevertheless proceeds unreasonably to make use of the product and is injured by it, he is barred from recovery." Madonna v. Harley Davidson page 586 * RULE: Evidence establishing an accident was solely the result of the user's conduct, and not related in any way to a product defect, is relevant and admissible to prove causation. * Contributory negligence applies only in situations when there is more than one proximate cause of the injury * The court finds that even if the bolt in the motorcycle was defective, the cause of the accident was the plaintiffs drunkenness * INFO: In Madonna, the court explained that because the evidence of the driver's conduct was offered to negate the theory that the defect caused the accident, and to establish that the driver's reckless conduct was its sole cause, such evidence was properly admitted. Some states have enacted products liability "reform" statutes providing various defenses based on a plaintiff's misconduct. The conduct is sometimes defined in terms of "reasonable care," which effectively reintroduces contributory negligence as a defense. Daly v. General Motors page 588 * Drunk, speeding, seatbeltless, Ps decedent who didnt lock the car door gets thrown out of an unlocked car door that pops open when he slammed into a guardrail * Negligence (P) v. Non-Negligence-Based Strict Liability (D) what you can do in these kind of cases is assess each partys relative contribution to the cause of the harm and also which party is in the best position to avoid the harm you basically just assess the reasonableness of each partys conduct * Some jurisdictions award recovery to the P, reduced based on Ps fault (what the court did in this case) * Some jurisdictions say that if the fault of the P is 50% or more, the P gets nothing * Problem w/ using comparative negligence * RULE: The principles of comparative negligence apply to actions founded on strict products liability. * INFO: The majority opinion asserted as an additional justification for its position the fact that, under prior law, assumption of the risk was a complete defense to an action for strict products liability. Under that system, "the curious and cynical message is that it profits the manufacturer to make his product so defective that in the event of injury he can argue that the user had to be aware of its patent defects to the extent the incentives are inverted." RST 3rd 17: * Ps recovery may be limited if Ps negligent conduct joined with the product defect in bringing about the harm to the P * Manner and extent of reduction is governed by generally applicable rules apportioning responsibility. General Motors v. Sanchez page 593 PRODUCTS LIABILITY 2009 Matthew J. Weaver Page 40

* RULE: Comparative responsibility applies in strict liability if a plaintiff `s negligence is something other than the mere failure to discover or guard against a product defect. * INFO: As the Sanchez case notes, the Restatement (Third) of Torts 17(a) provides that a plaintiff `s conduct, of whatever type, should be considered to reduce a damages recovery if it fails to conform to applicable standards of care. Comment d states that in general a plaintiff has no reason to expect that a new product contains a defect and would have little reason to be on guard to discover it. RST2d 402A, comment n: The form of contributory negligence which consists in voluntarily and unreasonably proceeding to encounter a known danger, and commonly passes under the name if assumption of risk, is a defense under this Section ASSUMPTION OF RISK Assumption of risk is a subjective test contributory negligence is an objective test Bowen v. Cochran page 603 * Assumption of risk case * D must present evidence that P had actual knowledge of the danger * D must present evidence Ps action to confront the risk was voluntary P wasnt acting on compulsion * D must present evidence that P appreciated the risk * ^^^ these are the classic 3 elements of assumption of risk * RULE: Evidence of assumption of risk is admissible where the plaintiff understands, appreciates, and voluntarily exposes himself to the risk in question. * INFO: As the Bowen case makes clear, to show assumption of risk, the defendant must present evidence that the plaintiff (1) had actual knowledge of the danger; (2) understood and appreciated the risk; and (3) voluntarily exposed himself to that risk. In this regard, an intelligent adult familiar with the product is more likely to be found to have assumed a risk than an inexperienced adult or a child. Moran v. Raymond page 598 * RULE: A plaintiff who knows a product is in a dangerous condition and proceeds in disregard of this known danger may not recover for resulting injuries. * INFO: The harshness of the assumption of risk doctrine counsels against its expansion or extension. Courts are loath to apply the doctrine to children or even to adults lacking experience with the product in question. Some jurisdictions require a plaintiff to be subjectively aware of the nature, character, and extent of the danger posed by the specific defect in question; a general awareness that the product is somehow dangerous is not sufficient. Johnson v. Clark page 606 * Knowledge of danger * Appreciation of risk * Voluntary decision to confront risk

PRODUCTS LIABILITY 2009 Matthew J. Weaver

Page 41

* In a Strict Liability case, the decision to the confront the risk must have been unreasonable * The court adds this one ^^^ * RULE: In a products liability action, a plaintiff will be barred from recovering if his decision to encounter the risk was unreasonable. * INFO: The court in this case noted that in determining whether the plaintiff's actions were reasonable, the decision to encounter a risk must be determined as of the time that the decision was made. In some jurisdictions, economic necessity, such as that demonstrated by this case, will not be an excuse for assuming a known hazard, unless protest by the worker would be futile. Assumption of risk is not assumption of the injury Assumption of risk is a complete defense to the plaintiffs claim Plaintiff recovers nothing if assumption of risk is found It is a complete bar to the plaintiffs recovery, just like misuse A partial defense is now available to strict liability defendants Should assumption of risk continue to be a complete defense? Assumption of risk can be built into a contributory negligence-style balancing test You cant always say that assumption of risk is the entire cause of an injury Defendant is sometimes in a better position to avoid harm in assumption of risk cases The trend is toward treating assumption of risk as part of the comparative inquiry This trend is far from uniform, however Week of April 13 SPECIAL TYPES OF DEFENDANTS 683-731 RETAILERS Zamora v. Mobil page 684 * Should we impose liability on a distributor? Well, they did make a profit on a product that killed 5 children. * RULE: Section 402A of Restatement (Second), by its literal terms, imposes strict liability for any sale of a defective product, and the sale of the product may trigger strict liability. * Here, Cal Gas (D) had no possession of the propane, but sold the gas completely as a paper transaction. However, Cal Gas (D) is properly regarded as a member in the chain of distribution of the propane. The primary policy justification recognized by this court for the extension of strict liability to all sellers in the chain of distribution is provision of the "maximum of protection" to the consumer. The sellers are then required to argue among themselves any questions as to their respective liability. Accordingly, Cal Gas (D) is appropriately included within the chain of distribution to Zamora (P). * INFO: Most courts have held that a retailer does not have a duty to test or inspect in the absence of notice that a product could be dangerous. However, the courts have also PRODUCTS LIABILITY 2009 Matthew J. Weaver Page 42

held that the retailer does have a duty to warn in certain circumstances. Such circumstances include where the retailer knows or should know that the product is likely to be dangerous when put to normal use. Reform statutes: Where the manufacturer is both solvent and subject to the jurisdiction of the court cant sue the retailer, gotta sue the manufacturer. One of the rationales is that we want to put the pressure on entities that are in a position to do something to make products safer. Retailers arent in the position to put pressure on manufacturers. But how about Walmart? This retailer has a lot of power. COMPONENT PART MANUFACTURERS Lee v. Butcher Boy page 691 * RULE: A component part manufacturer who has no role in designing the finished product and who supplied a nondefective part cannot be held liable for defective design. * The court says that there is nothing defective about the component, the defect only arises once the component is integrated no liability * Component manufacturer shouldnt have the burden of investigating every use their component is put to * It wasnt possible for the manufacturer of the component part to do anything * INFO: The court also held that the motor's manufacturer had no duty to warn because it gave no input and had no control over the design and packaging of the grinder. Courts are reluctant to impose component part liability where another manufacturer does the packaging, labeling, and marketing. [I might have missed a class] Apperson v. E.I. Du Pont De Nemours & Co. * RULE: A manufacturer of a safe, raw-material component is not liable for injury caused by the final product if the final assembly is what renders the product dangerous and the manufacturer warns the final product assembler of any nonobvious risks in the product usage. * INFO: This case tackles the issue of how responsible a raw component manufacturer should be in determining the dangers of potential end uses of its product. It is argued that imposing liability where the manufacturer was to some degree aware of the potential use of the material would promote ignorance on the part of the manufacturer, turning a blind eye to known risky uses to avoid liability. At least in this case, Du Pont (D) supplied Vitek with current scientific evaluations of a similar use of Teflon. Berkovitz v. United States * RULE: No liability shall lie, under the Federal Torts Claims Act, for any claim based upon the performance of a discretionary function on the part of a federal agency or employee, whether or not the discretion involved is abused.

PRODUCTS LIABILITY 2009 Matthew J. Weaver

Page 43

* INFO: The Federal Torts Claims Act generally authorizes suits against the United States for damages. Such damages include injury or loss of property, or personal injury or death caused by negligent or wrongful behavior on the part of a government employee. The injury must occur under circumstances where the United States, if a private person, would be liable to the claimant in accordance with the law of the place where the injury occurred. Kerstetter v. Pacific Scientific Co. * RULE: The selection of the appropriate design for military equipment to be used by the Armed Forces is a discretionary function giving rise to the government contractor defense. * INFO: As indicated in the Kerstetter case, the design ultimately selected may well reflect a significant policy judgment by government officials whether or not the contractor, rather than those officials, developed the design. Courts, such as the instant one, usually find it to be unsound policy to penalize, and thus deter, active contractor participation in the design process by placing the contractor at risk unless it can identify all design defects. Fletcher v. Atex, Inc. * RULE: (1) If a parent company observes corporate formality in its dealings with its subsidiary, and the subsidiary is financially able to operate independently, the parent corporation is not liable for the subsidiary's acts under an alter ego theory. (2) A parent corporation is not liable for the actions of its subsidiary, under agency theory, when the parent corporation has taken no action to indicate that the subsidiary acts as its agent. (3) A parent company that puts out as its own product a chattel manufactured by its independent subsidiary is subject to liability as though it were the actual manufacturer. * INFO: The apparent manufacturer theory arose from the concept of estoppel. By embracing a product as its own and inducing a purchaser to rely on the seller's reputation, the seller could not then claim that it was not liable as the manufacturer after so implying in the course of dealing. The doctrine becomes more important as corporate structures become increasingly complex. ENDORSERS Hanberry v. Hearst page 721 * Good Housekeeping Seal of Approval * Endorsers arent sellers * Its more attenuated for endorsers to try and shift losses than it would be for an actual seller they do make money from their endorsements though * RULE: An endorser who, for financial gain, induces the public to purchase a product that is defective and not as represented in the endorsement is liable to a purchaser relying on the endorsement. * INFO: As the opinion indicates in dicta, merely printing an advertisement in a magazine is not necessarily an endorsement of the type which gives rise to a representation of safety or fitness. Something further is required, which would reasonably PRODUCTS LIABILITY 2009 Matthew J. Weaver Page 44

give the public the impression that the publication is lending its identity and reputation to the fitness and safety of the product. SUCCESSOR CORPORATIONS To what extent will the successor corporation be liable for defective products sold by the predecessor corporation? If the transaction is structured as a merger between 1 and 2, the resulting merged entity will have the liabilities of both of the merged corporations. If the transaction is structured as a sale of assets, the successor corporation does not take on the liabilities predecessor corporation. Is the successor in the position to make changes to the Does the successor continue the product line? Semenetz v. Sherling page 723 * Exceptions to the general rule of no liability: 1) Successor expressly or impliedly assumes predecessors tort liability; 2) Consolidation or merger; 3) Successor is mere continuation of predecessor; 4) Fraudulent transaction to evade liability * RULE: The "product line" exception to successor nonliability does not promote either sound business interests for the community or a fair apportionment of products liability. * INFO: As the Semenetz court explains, the California Supreme Court's rationale for the "product line" exception is difficult to defend. Research on the ability of successor corporations to weather the expenses of successor liability contradicts the California court's confident assertion that a successor can assume such expenses. Moreover, imposing liability as a legal fiction instead of as a factual reality also contradicts the California court's stated concern for fairness. A majority of U.S. courts therefore follow a rationale similar to that of the Semenetz court in rejecting the "product line" exception to successor non-liability. * Exception for mere continuation takes place where you have a sale but the relationship that comes out after the sale, liking at the top line, who is actually running the corporation (shareholders, boards, management), the courts will not recognize it as a different corporation so if its the same product made at the same factory by the same people, then there is continuity of enterprise On another note: Employers as manufacturers: one way that employees have tried to get around the workers compensation is to bring suit when the company manufacturers or puts into final finished form products that will be used to make the products that the employees use to make the products that the manufacturer sells (haha) example is the press cases but it hasnt actually gone anywhere for some reason PREEMPTION When federal body regulates a product, and the product complies with these regulations, is it fair to allow a plaintiff to bring a case? PRODUCTS LIABILITY 2009 Matthew J. Weaver Page 45

The answer to this requires close statutory reading & the application of some doctrines that the S.C. has developed * Express preemption * Implied preemption - Conflict preemption - Impossibility preemption: it would be impossible for D to comply with both federal law and state law. - Purpose preemption: allowing the state lawsuit would serve as an unacceptable obstacle to achieving Congresss purpose or ends - Field preemption - Pervasiveness: federal regulation of the field is so complete and pervasive that it is reasonable to infer that Congress left no room for state law to supplement federal regulation - Interests: federal interests in the field are so dominant as to exclude state regulation in the same field Applying the actual doctrine of preemption is complex. This is because Congress drafts funny bills. Many bills have concurrent preemption clauses and savings clauses. Sometimes these can be reconciled it all depends on the language that is actually used. The law here is somewhat unstable! Riegel not in book *Medical Device Amendments Act of 1976: Except as otherwise provided, no state or political subdivision of a state may establish or continue in effect with respect to a device intended for human use any requirement 1) which is different from, or in addition to, any requirement applicable under this chapter to the device, and 2) which relates to the safety or effectiveness of the device. A state private law lawsuit something something these preemption cases are hard to understand. Some other case Stuff got injected into some lady Her arm fell off She was a musician D pointed to some place P pointed to some other place The court goes off onto some tangent Look at the structure of the lawsuit it is doing some stuff

PRODUCTS LIABILITY 2009 Matthew J. Weaver

Page 46

Potrebbero piacerti anche